Decide if the situation involves​ permutations, combinations, or neither. Explain your reasoning. Does the situation involve​ permutations, combinations, or​ neither? Choose the correct answer below. A. B. C. Neither. A line of people is neither an ordered arrangement of​ objects, nor a selection of objects from a group of objects.

Answers

Answer 1

Answer:

C. Neither

Step-by-step explanation:

The permutation is a selection of objects from a given sample in an ordered manner .

The combination is a selection of objects from a given sample irrespective of an order of arrangement.

The given line of people is neither an ordered arrangement of​ objects, nor a selection of objects from a group of objects So it fits neither of the combinations or permutations.

So the best answer is neither.


Related Questions

A parent increases a child’s monthly allowance by 20% each year. If the allowance is $8 per month now, in about how many years will it take to reach $20 per month? Use the equation 20 = 8(1.2)x to solve the problem. Round to the nearest year. 1 year 5 years 2 years 16 years

Answers

Answer:

6 years

Step-by-step explanation:

A parent increases a child’s monthly allowance by 20% each year. If the allowance is $8 per month now. This is an exponential function, An exponential function is given by:

[tex]y=ab^x[/tex]

Let x be the number of years and y be the allowance. The initial allowance is $8, this means at x = 0, y = 8

[tex]y=ab^x\\8=ab^0\\a=8[/tex]

Since it increases by 20% each year, i.e 100% + 20% = 1 + 0.2 = 1.2. This means that b = 1.2

Therefore:

[tex]y=ab^x\\y=8(1.2^x) \\[/tex]

To find the number of years will it take to reach $20 per month, we substitute y = 20 and find x

[tex]20=8(1.2)^x\\20/8=1.2^x\\1.2^x=2.5\\Taking \ natural\ log\ of \ both\ sides:\\ln(1.2^x)=ln2.5\\xln(1.2)=0.9163\\x=0.9163/ln(1.2)\\x=5.026[/tex]

x = 6 years to the nearest year

Answer:

5 years

Step-by-step explanation:444

Determine if the process appears to be within statistical control. If not, state the reason why not.
a. It does not appear to be within statistical control because there is an upward shift.
b. It appears to be within statistical control.
c. It does not appear to be within statistical control because there is an upward trend.
d. It does not appear to be within statistical control because there is increasing variation.

Answers

Answer:

c. It does not appear to be within statistical control because there is an upward trend.

Step-by-step explanation:

Statistical process control is a method for quality control which employs statistical method to monitor and control process. It ensures operation efficiency and ensuring required specification to reduce wastes in production lines. Here the process variation is out of control because the statistical control has an upward trend.


What is the midline equation of the function h(x) = -4 cos(5x - 9) - 7?

Answers

Answer: Midline equation: y = -7

Step-by-step explanation: This function is a sinusoidal function of the form:

y = a.cos(b(x+c))+d

Midline is a horizontal line where the function oscillates above and below.

In the sinusoidal function d represents its vertical shift. Midline is not influenced by any other value except vertical shift. For that reason,

Midline, for the function: [tex]h(x) = -4cos(5x-9) - 7[/tex] is y=d, i.e., [tex]y=-7[/tex]

Answer:

y=-7

Step-by-step explanation:

PLEASE HELP ! (2/5) -50 POINTS -

Answers

Answer:

symmetric

Step-by-step explanation:

it kind of evenly falls to the left and right from the highest value in the middle

skewed would be different and would look like a straight line not a quadratic equation

C) symmetric distribution because the mean, median, mode happen at about the same point

Time spent using e-mail per session is normally distributed with a mean = to 8 minutes and standard deviation = 2minutes. If a random samples of 36 sessions were selected, the computed sample standard deviation would be

a. 0.25
b. 0.3333
c. 0.42
d. 0.48

Answers

Answer:

The correct option is (b) 0.3333.

Step-by-step explanation:

The standard deviation of the sampling distribution of sample mean [tex](\bar x)[/tex] is known as the standard error [tex](\sigma_{\bar x})[/tex].

The standard error is given as follows:

[tex]\sigma_{\bar x}=\frac{\sigma}{\sqrt{n}}[/tex]

The information provided is:

[tex]\mu=8\\\\\sigma=2\\\\n=36[/tex]

Compute the standard deviation of the sample mean as follows:

[tex]\sigma_{\bar x}=\frac{\sigma}{\sqrt{n}}[/tex]

    [tex]=\frac{2}{\sqrt{36}}\\\\=\frac{2}{6}\\\\=\frac{1}{3}\\\\=0.3333[/tex]

Thus, the standard deviation of the sample mean is 0.3333.

Given the following hypotheses: H0: μ = 490 H1: μ ≠ 490 A random sample of 15 observations is selected from a normal population. The sample mean was 495 and the sample standard deviation 9. Using the 0.01 significance level:
a.) State the decision rule.
b.) Compute the value of the test statistic.
c.) What is your decision regarding the null hypothesis?

Answers

Answer:

We conclude that the population mean is equal to 490.

Step-by-step explanation:

We are given that a random sample of 15 observations is selected from a normal population. The sample mean was 495 and the sample standard deviation 9.

Let [tex]\mu[/tex] = population mean.

So, Null Hypothesis, [tex]H_0[/tex] : [tex]\mu[/tex] = 490      {means that the population mean is equal to 490}

Alternate Hypothesis, [tex]H_A[/tex] : [tex]\mu\neq[/tex] 490     {means that the population mean is different from 490}

The test statistics that will be used here is One-sample t-test statistics because we don't know about population standard deviation;

                               T.S.  =  [tex]\frac{\bar X-\mu}{\frac{s}{\sqrt{n} } }[/tex]  ~ [tex]t_1_4[/tex]

where, [tex]\bar X[/tex] = sample mean = 495

            s = sample standard deviation = 9

             n = sample of observations = 15

So, the test statistics =   [tex]\frac{495-490}{\frac{9}{\sqrt{15} } }[/tex]  ~ [tex]t_1_4[/tex]

                                     =  2.152

The value of t-test statistics is 2.152.

Now, at a 0.01 level of significance, the t table gives a critical value of -2.977 and 2.977 at 14 degrees of freedom for the two-tailed test.

Since the value of our test statistics lies within the range of critical values of t, so we have insufficient evidence to reject our null hypothesis as the test statistics will not fall in the rejection region.

Therefore, we conclude that the population mean is equal to 490.

normal population has a mean of 63 and a standard deviation of 13. You select a random sample of 25. Compute the probability that the sample mean is: (Round your z values to 2 decimal places and final answers to 4 decimal places): Greater than 65.

Answers

Answer:

0.2207

Step-by-step explanation:

Here, we want to find the probability that the sample mean is greater than 25.

What we use here is the z-scores statistic

Mathematically;

z-score = (x-mean)/SD/√n

From the question;

x = 65, mean = 63, SD = 13 and n = 25

Plugging these values in the z-score equation, we have

Z-score = (65-63)/13/√25 = 2/13/5 = 0.77

So the probability we want to calculate is ;

P(z > 0.77)

This can be obtained from the standard normal distribution table

Thus;

P(z > 0.77) = 0.22065 which is 0.2207 to 4 d.p

solve the following inequalities 7 x minus 5 / 8 x + 3 >4

Answers

Answer:

[tex]x> \frac{8}{51} [/tex]

Step-by-step explanation:

[tex]7x - \frac{5}{8} x + 3>4[/tex]

Bring constants to one side, simplify:

[tex] \frac{51}{8} x>4 - 3 \\ \frac{51}{8} x>1 \\ x>1 \div \frac{51}{8} \\ x>1 \times \frac{8}{51} \\ x> \frac{8}{51} [/tex]

*Note that the inequality sign only changes when you divide the whole inequality by a negative number.

Answer:

[tex]x>\frac{8}{51}[/tex]

Step-by-step explanation:

[tex]7x-\frac{5}{8}x+3>4\\\mathrm{Subtract\:}3\mathrm{\:from\:both\:sides}\\7x-\frac{5}{8}x+3-3>4-3\\\mathrm{Simplify}\\7x-\frac{5}{8}x>1\\\mathrm{Multiply\:both\:sides\:by\:}8\\7x\times \:8-\frac{5}{8}x\times \:8>1\times \:8\\\mathrm{Simplify}\\56x-5x>8\\51x>8\\\mathrm{Divide\:both\:sides\:by\:}51\\\frac{51x}{51}>\frac{8}{51}\\\\x>\frac{8}{51}[/tex]

I hope it helps :)



x = 4: 3x3 - 2x2 +10

Answers

Answer:

170

Step-by-step explanation:

3(4)³ - 2(4)² + 10

192 - 32 + 10 = 170

if G is the midpoint of FH, FG = 14x + 25 and GH = 73 - 2x, find FH.

Answers

Answer:

FH = 134

Step-by-step explanation:

From the question given:

G is the midpoint of FH

FG = 14x + 25

GH = 73 - 2x

FH =?

Next, we shall determine the value of x. The value of x can be obtained as follow:

Since G is the midpoint of FH, this implies that FG and GH are equal i.e

FG = GH

With the above formula, we can obtain the value of x as follow:

FG = 14x + 25

GH = 73 - 2x

x =?

FG = GH

14x + 25 = 73 - 2x

Collect like terms

14x + 2x = 73 - 25

16x = 48

Divide both side by 16

x = 48/16

x = 3

Next, we shall determine the value of FG and GH. These can be obtained as shown below:

FG = 14x + 25

x = 3

FG = 14x + 25

FG = 14(3) + 25

FG = 42 + 25

FG = 67

GH = 73 - 2x

x = 3

GH = 73 - 2x

GH = 73 - 2(3)

GH = 73 - 6

GH = 67

Finally, we shall determine FH as follow:

FH = FG + GH

FG = 67

GH = 67

FH = FG + GH

FH = 67 + 67

FH = 134

Therefore, FH is 134

una compañía sabe que si produce "x" unidades mensuales su utilidad "u" se podría calcular con la expresión:


u(x)=-0.04x^2+44x-4000


donde "u" se expresa en dólares. Determine la razón del cambio promedio de la utilidad cuando el nivel de producción cambia de 600 a 620 unidades mensuales. Recuerde que la pendiente de la recta secante a la gráfica de la función representa a la razón de cambio promedio.


porfavor alguien que me explique el procedimiento :(

Answers

Answer:

Δf(u) /Δx  = 92,8    ( razón de cambio promedio)

Step-by-step explanation:

La expresión de la utilidad de la empresa u(x) en función de la cantidad de unidades producidas "x" es:

u(x) = 0,04*x² + 44*x  -4000

Entonces la razón de cambio promedio en un intervalo (a ; b)  en este caso ( 620 ;  600 ) viene dada por la expresión:

Δf(x)/ Δx  =  [ f(b)  -  f(a) ]/( b - a )

en donde  f(b)  y f(a) se obtienen por sustitución de los valores a y b es decir  600 y 620 respectivamente en la función f(x) = u(x) entonces

Δf(u) /Δx  =  [ u(b)  -  u(a) ]/( b -a )        (1)

u(b) =  0,04*(620)² + 44*(620) - 4000

u(b) = 15376 + 27280 -4000

u(b) = 2656 unidades

u(a)  = 0,04* (600)²  +  44* 600  - 4000

u(a)  = 14400  +  26400  - 4000

u(a) = 800

Sustituyendo esos valores en la ecuación 1

Δf(u) /Δx  =  2656 - 800 / 620 - 600

Δf(u) /Δx  = 92,8

What is the exact distance from (−1, 4) to (6, −2)? square root of 80. units square root of 82. units square root of 85. units square root of 89. units

Answers

Answer:

[tex]\sqrt{85}[/tex].

Step-by-step explanation:

[tex]x[/tex]-coordinates:

First point: [tex]-1[/tex].Second point: [tex]6[/tex].Difference: [tex]|-1 - 6| = |-7| = 7[/tex].

[tex]y[/tex]-coordinates:

First point: [tex]4[/tex].Second point: [tex]-2[/tex].Difference: [tex]|4 - (-2)| = |6| = 6[/tex].

Refer to the diagram attached. Consider these two points as the two end points of the hypotenuse of a right triangle. The lengths of the two legs are equal to:

the difference between the two [tex]x[/tex]-coordinates, [tex]7[/tex], and the difference between the two [tex]y[/tex]-coordinates, [tex]6[/tex].

Apply Pythagorean Theorem to find the length of the hypotenuse (which is equal to the distance between the two points in question.)

[tex]\begin{aligned}\text{Hypotenuse} &= \sqrt{(\text{First Leg})^2 + (\text{Second Leg})^2} \\ &= \sqrt{7^2 + 6^2} \\ &= \sqrt{85}\end{aligned}[/tex].

Answer:

C

Step-by-step explanation:

A new fast-food firm predicts that the number of franchises for its products will grow at the rate dn dt = 6 t + 1 where t is the number of years, 0 ≤ t ≤ 15.

Answers

Answer:

The answer is "253"

Step-by-step explanation:

In the given- equation there is mistype error so, the correct equation and its solution can be defined as follows:

Given:

[tex]\bold{\frac{dn}{dt} = 6\sqrt{t+1}}\\[/tex]

[tex]\to dn= 6\sqrt{t+1} \ \ dt.....(a)\\\\[/tex]

integrate the above value:

[tex]\to \int dn= \int 6\sqrt{t+1} \ \ dt \\\\\to n= \frac{(6\sqrt{t+1} )^{\frac{3}{2}}}{\frac{3}{2}}+c\\\\\to n= \frac{(12\sqrt{t+1} )^{\frac{3}{2}}}{3}+c\\\\[/tex]

When the value of n=1 then t=0

[tex]\to 1= \frac{12(0+1)^{\frac{3}{2}}}{3}+c\\\\ \to 1= \frac{12(1)^{\frac{3}{2}}}{3}+c\\\\\to 1-\frac{12}{3}=c\\\\\to \frac{3-12}{3}=c\\\\\to \frac{-9}{3}=c\\\\\to c=-3\\[/tex]

so the value of  n is:

[tex]\to n= \frac{(12\sqrt{t+1} )^{\frac{3}{2}}}{3}-3\\\\[/tex]

when we put the value t= 15 then,

[tex]\to n= \frac{(12\sqrt{15+1} )^{\frac{3}{2}}}{3}-3\\\\\to n= \frac{(12\sqrt{16} )^{\frac{3}{2}}}{3}-3\\\\\to n= \frac{(12\times 64)}{3}-3\\\\\to n= (4\times 64)-3\\\\\to n= 256-3\\\\\to n= 253[/tex]

What is the sum of the geometric sequence?

Answers

Answer:

B. 259

Step-by-step explanation:

6^(i - 1) for i = 1 to 4

sum = 6^(1 - 1) + 6^(2 - 1) + 6^(3 - 1) + 6^(4 - 1) =

= 6^0 + 6^1 + 6^2 + 6^3

= 1 + 6 + 36 + 216

= 259

Answer: B. 259

a lottery offers one $1000 prize one $500 and two $50 prizes. one thousand tickets are sold at $2.50. what is the expectived profit

Answers

Answer:

 $900

Step-by-step explanation:

To begin with let us estimate the total cash value of the prices

$1000 x 1= 1000

$500 x 1=  500

$50 x 2= 100

Total = $1600

Now let us calculate the total cost of tickets sold at $2.50 per tickets for 1000 tickets

2.5*1000= $2,500

Assuming worse case that the lottery had winners in all three categories and i.e the total prices given out is $1600

Then the expected profit is = $2,500-$1600= $900

Please! David has several chains of length 5 and of length 7. By joining chains one after the other, David can create different lengths. Which of these lengths is impossible to make? A)10 B)12 C)13 D)14 E)15

Answers

Answer:

13

Step-by-step explanation:

A)5+5=10

B)5+7=12

C) impossible

D)7+7=14

E)5+5+5=15

Geometry pls help !!! Find the value of AB.
AB = [?]

Answers

Answer:

AB = 16 Units

Step-by-step explanation:

In the given figure, CD is the diameter and AB is the chord of the circle.

Since, diameter of the circle bisects the chord at right angle.

Therefore, AE = 1/2 AB

Or AB = 2AE...(1)

Let the center of the circle be given by O. Join OA.

OA = OD = 10 (Radii of same circle)

Triangle OAE is right triangle.

Now, by Pythagoras theorem:

[tex] OA^2 = AE^2 + OE^2 \\

10^2 = AE^2 + 6^2 \\

100= AE^2 + 36\\

100-36 = AE^2 \\

64= AE^2 \\

AE = \sqrt{64}\\

AE = 8 \\

\because AB = 2AE..[From \: equation\: (1)] \\

\therefore AB = 2\times 8\\

\huge \purple {\boxed {AB = 16 \: Units}} [/tex]

One of two small restaurants is chosen at random with equally likely probability, and then an employee is chosen at random from the chosen restaurant. Restaurant #1 has 10 full-timers and 6 part-timers. Restaurant #2 has 7 full-timers and 9 part-timers. What is the probability that Restaurant #1 was chosen at random, given that a full-time employee was chosen? Your answers should be rounded to 4 digits after the decimal.

Answers

Answer:

P(1 |F) = 10/17

Step-by-step explanation:

Let events

1 = restaurant 1

2 = restaurant 2

F = full-time worker chosen

P = part-time worken chosen

P(1 and F) = 1/2 * 10/16 = 5/16

P(2 and F) = 1/2 * 7/16 = 7/32

P( (1 or 2) and F ) = P(F) = 5/16+7/32 = 17/32

P(1 | F)          Probability of choosing restaurant 1 given a full-time was chosen

= P(1 and F) / P(F)

= 5/16  / (17/32)

= 5/16 * 32/17

= 10 / 17

What is the constant of variation, k, of the line y=kx through (3,18) and (5,30)? 3 6

Answers

Answer:

6

Step-by-step explanation:

The constant of variation is the slope

k = (y2-y1)/(x2-x1)

  = (30-18)/(5-3)

   =12/2

   = 6

The value of constant of variation, k, is,

⇒ k = 6

What is Equation of line?

The equation of line in point-slope form passing through the points

(x₁ , y₁) and (x₂, y₂) with slope m is defined as;

⇒ y - y₁ = m (x - x₁)

Where, m = (y₂ - y₁) / (x₂ - x₁)

Here, the constant of variation, k, of the line y = kx through (3,18) and (5,30)

Since, The constant of variation is the slope,

Hence, We get;

k = (y₂ - y₁)/(x₂ - x₁)

 = (30 - 18)/(5 - 3)

  = 12/2

  = 6

Thus, the value of constant of variation, k, is,

⇒ k = 6

Learn more about the equation of line visit:

https://brainly.com/question/18831322

#SPJ7

(a) A survey of the adults in a town shows that 8% have liver problems. Of these, it is also found that 25% are heavy drinkers, 35% are social drinkers and 40% are non-drinkers. Of those that did not suffer from liver problems, 5% are heavy drinkers, 65% are social drinkers and 30% do not drink at all. An adult is chosen at random, what is the probability that this person i. Has a liver problems?

Answers

Answer:

The probability that the selected adult has liver problems is 0.08

Step-by-step explanation:

In this question, from the data given, we want to calculate the probability that an adult selected at random has liver problems.

Let E(L) be the event that an adult has liver problems.

The probability is directly obtainable from the question and it is given as 8%

Thus, the probability that the selected adult has liver problems; P(L) = 8% = 8/100 = 0.08

Use the frequency distribution, which shows the number of American voters (in millions) according to age, to

find the probability that a voter chosen at random is in the 18 to 20 years old age range

Ages

18 to 20

21 to 24

25 to 34

35 to 44

45 to 64

65 and over

Frequency

4.2

7.8

20.8

23.7

50.1

28 2

Date

07/2

3:29

The probability that a voter chosen at random is in the 18 to 20 years old age range is 0.0311

(Round to three decimal places as needed.)

07/2

8:52

Question Viewer

07/1

8:03

07/1

5:46

>

07/1

12:2

07/1

5:39

07/1

2:42

Question is complete. Tap on the red indicators to see incorrect answers.

07/1

12:00

Answers

Answer:

The probability that a voter chosen at random is in the 18 to 20 years old age range is =  4.2/ 134.8= 0.0311572

Step-by-step explanation:

We can find the probability by simply dividing the frequency of the ages range of 18-20 by the total frequency.

Ages                Frequency

18 to 20             4.2

21 to 24             7.8

25 to 34           20.8

35 to 44           23.7

45 to 64           50.1

65 and over       28.2

∑                        134.8

The probability of an event is given by the occurrence of an event by the total occurrences .

So

Here the occurrence of ages 18-20 is given by 4.2

and the total frequency is 134.8

The probability that a voter chosen at random is in the 18 to 20 years old age range is =  4.2/ 134.8= 0.0311572

Translate and solve: 54 greater than x is greater than 216

Answers

Answer:

x >162

Step-by-step explanation:

x+54 > 216

Subtract 54 from each side

x+54-54 > 216 - 54

x >162

Answer:

[tex]\huge \boxed{{x>162}}[/tex]

Step-by-step explanation:

[tex]x+54 > 216[/tex]

[tex]\sf Subtract \ 54 \ from \ both \ parts.[/tex]

[tex]x+54 -54> 216-54[/tex]

[tex]x>162[/tex]

Yelena needs to swim a total of 8 miles this
week. So far, she swam 5 miles. Use the
equation 5 + m=8 to find how many more
miles Yelena needs to swim.

Answers

Answer:

3 miles

Step-by-step explanation:

5 + m=8

Subtract 5 from each side

5-5 + m=8-5

m = 3

She needs to swim 3 more miles

Answer:

Yelena needs to swim 3 more miles

Step-by-step explanation:

You need to solve for the variable "m", which represents the miles. Based on the information, Yelena swam 5 miles and she needs to swim 8. Solve:

[tex]5+m=8[/tex]

To find the value of m, you need to isolate it on one side of the equation. To do this, you need to get the 8 and 5 on the same side of the equal operation. For this, you need to use reverse operations. This undoes the value from one side and does the same on the other, keeping the equation balanced. Since we have a "positive 5", we take the opposite, which would be a "negative 5". So subtract 5 from both sides of the equation:

[tex]5-5+m=8-5[/tex]

Simplify. The 5's cancel each other out, leaving 0. 8-5 is 3:

[tex]m=3[/tex]

The total miles left that Yelena needs to swim is 3 miles.

:Done

Find an exact value of sin(17pi/12)

Answers

●✴︎✴︎✴︎✴︎✴︎✴︎✴︎✴︎❀✴︎✴︎✴︎✴︎✴︎✴︎✴︎✴︎✴︎●

   Hi my lil bunny!

❧⎯⎯⎯⎯⎯⎯⎯⎯⎯⎯⎯⎯⎯⎯⎯⎯⎯⎯⎯⎯⎯⎯⎯⎯⎯⎯⎯⎯⎯⎯⎯⎯⎯⎯⎯⎯⎯⎯☙

[tex]\frac{(17)(3.141593)}{12}[/tex]

= [tex]\frac{53.407075}{12}[/tex]

= [tex]4.45059[/tex]

❧⎯⎯⎯⎯⎯⎯⎯⎯⎯⎯⎯⎯⎯⎯⎯⎯⎯⎯⎯⎯⎯⎯⎯⎯⎯⎯⎯⎯⎯⎯⎯⎯⎯⎯⎯⎯⎯⎯☙

●✴︎✴︎✴︎✴︎✴︎✴︎✴︎✴︎❀✴︎✴︎✴︎✴︎✴︎✴︎✴︎✴︎✴︎●

If this helped you, could you maybe give brainliest..?

Also Have a great day/night!

❀*May*❀

I have an answer and explanation but I can't type so search up the question you asked and you should get an answer and explanation from s0cratic.

Which of the following is true about congruent figures?
They're the same shape and the same size.
They're the same size, but not the same shape.
They're not the same shape or size.
They're the same shape, but not the same size.​

Answers

Answer:

A

Step-by-step explanation:

congruent means they have the same shape and size. hope this helps :)

1. What is the value of (1/2)^3?
O A. 76
O B. 119
O C.12
O D. 18​

Answers

Answer:

1/2 to the power of 3= 1/8

Step-by-step explanation:

1/2*1/2=1/4

1/4*1/2=1/8

d?

[tex]\huge\text{Hey there!}[/tex]


[tex]\mathsf{(\dfrac{1}{2})^3}[/tex]

[tex]\mathsf{= \dfrac{1}{2}^3}[/tex]

[tex]\mathsf{= \dfrac{1}{2} \times \dfrac{1}{2} \times \dfrac{1}{2}}[/tex]

[tex]\mathsf{= \dfrac{1 \times 1 \times 1}{2 \times 2 \times 2}}[/tex]

[tex]\mathsf{\mathsf{= \dfrac{1 \times 1} {4 \times 2}}}[/tex]

[tex]\mathsf{= \dfrac{1}{8}}[/tex]



[tex]\huge\text{Therefore your answer should be:}[/tex]

[tex]\huge\boxed{\mathsf{Option\ D.\ \dfrac{1}{8}}}\huge\checkmark[/tex]


[tex]\huge\text{Good luck on your assignment \& enjoy your day!}[/tex]



~[tex]\frak{Amphitrite1040:)}[/tex]

coefficient of 8x+7y

Answers

Answer: I’m pretty sure it is 8 and 7 because those are the numbers multiplied with variables.

Answer:

8

Step-by-step explanation:

Identify the exponents on the variables in each term, and add them together to find the degree of each term.

8x→1

7y→1

The largest exponent is the degree of the polynomial.

1

The leading term in a polynomial is the term with the highest degree.

8x

The leading coefficient of a polynomial is the coefficient of the leading term.

____________________________________________________________

The leading term in a polynomial is the term with the highest degree.

8x

The leading coefficient in a polynomial is the coefficient of the leading term.

8

List the results.

Polynomial Degree: 1

Leading Term: 8x

Leading Coefficient: 8

Hope This Helps!!!

The following data represents the age of 30 lottery winners.

22 26 27 27 31 34
36 42 43 44 48 49
52 53 55 56 57 60
65 65 66 67 69 72
75 77 78 78 79 87
Complete the frequency distribution for the data.

Age Frequency
20-29
30-39
40-49
50-59
60-69
70-79
80-89

Answers

Answer:

Step-by-step explanation:

This is an example of a frequency distribution for a class interval. In order to complete the frequency distribution, we will count the number of data occurring in each group, and write that number as the frequency for that group. This is done as shown below:

 Age                Frequency          ages in class

20-29                       4                  22, 26, 27, 27                

30-39                       3                  31, 34, 36

40-49                       5                  42, 43, 44, 48, 49

50-59                       5                  52, 53, 55, 56, 57

60-69                       6                  60, 56, 65, 66, 67, 69

70-79                        6                  72, 75, 77, 78, 78, 79

80-89                       1                   87

Total                        30

WILL GIVE BRAINLEST PLEASE!!!!!!!! Jenny has some tiles in a bag. The tiles are of three different colors: purple, pink, and orange. Jenny randomly pulls a tile out of the bag, records the color, and replaces the tile in the bag. She does this 50 times. The results are recorded in the given table: Color of Tile Purple Pink Orange Number of times the tile is drawn 6 18 26 What is the experimental probability that Jenny will pull out a purple tile? fraction 6 over 50 fraction 44 over 50 fraction 6 over 44 fraction 18 over 44

Answers

Answer:

6/50

Step-by-step explanation:

There are 50 tiles

6 purple

18 pink

26 orange

P( purple) = purple/ total

                = 6/50

Which is a factor of: 2x2+10x+8 ?

Answers

Answer:

2 ( x+4) ( x+1)

Step-by-step explanation:

2x^2+10x+8

Factor out 2

2 ( x^2 +5x+4)

What two numbers  multiply to 4 and add to 5

4*1 = 4

4+1 = 5

2 ( x+4) ( x+1)

[tex] \large{ \underline{ \underline{ \bf{ \pink{To \: factorise}}}}}[/tex]

2x² + 10x + 8

Factorisation:

By middle term factorisation,

⇛ 2x² + 2x + 8x + 8

⇛ 2x(x + 1) + 8(x + 1)

⇛ (2x + 8)(x + 1)

⇛ 2(x + 4)(x + 1)

☃️ Now you can break it down and check which are the factors of the polynomial according to options.

━━━━━━━━━━━━━━━━━━━━

Other Questions
Find the average magnitude of the induced emf if the change in shape occurs in 0.125 ss and the local 0.504-TT magnetic field is perpendicular to the plane of the loop. A shop sells DVDs and CDs.DVDs are sold at one price. CDs are sold at a different price.2 DVDs and 1 CD cost 35 2 DVDs and 2 CDs cost 45 Martin has 50 Does he have enough to buy 1 DVD and 3 CDs? Columbus first landed at:HispaniolaCubaJamaicaSan Salvador The study of such concepts as motion, force, energy, matter, heat, sound, light, and the components of atoms is: Problem 24-01 Liquidation Southwestern Wear Inc. has the following balance sheet: Current assets $1,875,000 Accounts payable $375,000 Fixed assets 1,875,000 Notes payable 750,000 Subordinated debentures 750,000 Total debt $1,875,000 Common equity 1,875,000 Total assets $3,750,000 Total liabilities and equity $3,750,000 The trustee's costs total $276,250, and the firm has no accrued taxes or wages. Southwestern has no unfunded pension liabilities. The debentures are subordinated only to the notes payable. If the firm goes bankrupt and liquidates, how much will each class of investors receive if a total of $4 million is received from sale of the assets? Distribution of proceeds on liquidation: 1. Proceeds from sale of assets $ 2. First mortgage, paid from sale of assets $ 3. Fees and expenses of administration of bankruptcy $ 4. Wages due workers earned within 3 months prior to filing of bankruptcy petition $ 5. Taxes $ 6. Unfunded pension liabilities $ 7. Available to general creditors $ Distribution to general creditors: Claims of General Creditors Claim (1) Application of 100% Distribution (2) After Subordination Adjustment (3) Percentage of Original Claims Received (4) Notes payable $ $ $ % Accounts payable $ $ $ % Subordinated debentures $ $ $ % Total $ $ $ The remaining $ will go to the common stockholders. LCM of x2+5x+6 and x2-x-6 is A high-voltage powerline operates at 500000 V-rms and carries an rms current of 500 A. If the resistance of the cable is 0.050/km, what is the resistive power loss in 200 km of the powerline? Sulfur dioxide reacts with oxygen to form sulfur trioxide. What change in hybridization of the sulfur occurs in this reaction ? g A class is debating pros and cons of using sea walls to reduce beach erosion. What is the best way for them inform theirdecision-making?They should each write an opinion paragraph to send to the local paper.They should hold the debate on stage, so that they can be judged.They should watch movies about beaches.They should research wave action and collect data about erosion in the area. Basic math for 20 points + brainliest! What is 12.5% of 72 Macbeth act 2 scene 1 The day before an exam, Jacob spent 2 1/5hours studying in the morning and 3 3/4 hoursstudying in the afternoon. How many hoursdid Jacob spend studying that day? According to myth, where did the Kiowa people come from? A. They sprang from the thought of the sun god. B. They broke away from the Crow people after an argument. C. They formed out of clay in the Yellowstone River. D. They entered the world through a hollow log. 1: The best statement for reason 6 of this proof is -A C -B D -B and D are supplements -B B 2.The best reason for statements 3.5. and 7 in this proof is - Alternate interior angles are congruent. -Corresponding angles are congruent. -Alternate exterior angles are congruent. -Interior angles on the same sides of a transversal are supplements. 3. The best statement for reason 8 of this proof is -B B -A and C are supplements. -B D -A C Arrange the sentences in order to describe how oxygen from air is transported to the cells in the kidneys. 23. The measure of an angle is 46. The measure ofthe complement isThe measure of thesupplement is.help if you can Assume Division 1 of the XYZ Company had the following results last year. Sales $5,000,000 Operating income 1,000,000 Total assets (average) 10,000,000 Current liabilities 500,000 Management's required rate of return is 8% and the weighted average cost of capital is 6%. Its effective tax rate is 30%. What is the division's economic value added? I need help on both answers. Theyre different from my other problems so Im kinda confused For the coming year, Belton Company estimates fixed costs of $60,000, the unit variable cost of $25, and the unit selling price of $50. a. Determine the break-even point in units of sales. b. Determine the unit sales required to realize operating income of $100,000. c. Determine the probable operating income if sales total $400,000.